Quick Calculus Midterm Question: Understanding Simplification Errors

  • Thread starter Thread starter justine411
  • Start date Start date
Click For Summary
The discussion revolves around a calculus midterm where the student lost points due to simplification errors in logarithmic expressions. The student correctly simplified an expression to pi/4 - 1/2ln3 but was marked down because the grader incorrectly added a 2 over the 3 in ln3, likely misunderstanding logarithmic properties. The student clarifies that ln1 equals zero, which should not contribute to the final expression. Additionally, the student questions the grading of their final simplification to pi/4 - ln(3^1/2), which is also deemed correct. The overall consensus is that the grading error stemmed from a misunderstanding of logarithmic rules, and the student hopes to have their mark adjusted.
justine411
Messages
16
Reaction score
0
I just got my calculus midterm back, and I lost one mark on an area problem. I did the problem properly, but I may have made a mistake at the end.

I end up with arctan1-1/2ln3-1/2ln1 (this step did not have an "x")
I simplified this to pi/4-1/2ln3 (here, the grader put a 2 over my 3 in ln3)
then I made it pi/4-ln(3^1/2) (there was a question mark around this)

so my quick questions are:

why was there are 2 over my three? (I thought 1/2ln1 was equal to zero.)
why was there a question mark around the last part (I thought alnx was equal to ln(x^a)

Hopefully this makes sense, post and I can clarify :biggrin:

thanks SO much in advance if you helped!
 
Physics news on Phys.org
It appears to me that the marker is unsure on properties of logarithms--you are correct.

The correct properties are: loga-logb=log(a/b) (1) and alogb=log(ba).

I end up with arctan1-1/2ln3-1/2ln1 (this step did not have an "x")
I simplified this to pi/4-1/2ln3 (here, the grader put a 2 over my 3 in ln3)
I can only think the marker has thought that ln3-ln1=ln(3-1)=2.* This is not correct. Of course, ln1=0. You can simplify the expression using (1) to give ln3-ln1=ln(3/1)=ln3.
then I made it pi/4-ln(3^1/2) (there was a question mark around this)

Again, this is correct.
____________
edit:*In retrospect, I don't see where the marker got ln2 from, since the expression would be -1/2{ln3+ln1} which, would become -1/2{ln4} using the incorrect rule. Anyway, the fact remains that you are correct.
 
Last edited:
Ok thanks! That's what I thought...hopefully I'll be able to get my prof to change my mark!
 
Question: A clock's minute hand has length 4 and its hour hand has length 3. What is the distance between the tips at the moment when it is increasing most rapidly?(Putnam Exam Question) Answer: Making assumption that both the hands moves at constant angular velocities, the answer is ## \sqrt{7} .## But don't you think this assumption is somewhat doubtful and wrong?

Similar threads

  • · Replies 3 ·
Replies
3
Views
2K
  • · Replies 5 ·
Replies
5
Views
2K
Replies
5
Views
1K
  • · Replies 5 ·
Replies
5
Views
2K
  • · Replies 9 ·
Replies
9
Views
2K
  • · Replies 6 ·
Replies
6
Views
2K
  • · Replies 2 ·
Replies
2
Views
3K
Replies
4
Views
2K
  • · Replies 3 ·
Replies
3
Views
2K
  • · Replies 1 ·
Replies
1
Views
2K